- PowerScore Staff
- Posts: 5972
- Joined: Mar 25, 2011
- Fri Jan 21, 2011 12:00 am
#41354
Complete Question Explanation
(The complete setup for this game can be found here: lsat/viewtopic.php?t=2035)
The correct answer choice is (D)
If G does not work during the week, then from the last rule we know that H cannot work during the week. Hence, only F, I, and J can work, and the only available numerical distribution is the 2-2-1. Despite this powerful information, and the application of the fourth rule, this is still a difficult question. If you make some good inferences, but still do not see an answer choice that is immediately and obviously correct, try to use hypotheticals to solve the problem. In this instance, answer choices (A) and (C) can both be eliminated with the following hypothetical:
Note that a savvy student could use the hypothetical created in question #7 answer choice (A)—which is the same as the hypothetical above—to eliminate answer choices (A) and (C) in question #11 (remember, question #7 is a Cannot Be True question, so the four wrong answer choices are viable solutions).
Answer choices (B) and (E) can both be eliminated with the following hypothetical:
Answer choice (D) is thus proven correct by process of elimination.
After reviewing the above information, some students still wonder why it must be that I works twice during the work week. If I only works once and F and J each work twice, then F and J are forced to “overlap” in a way that violates the fourth rule (to do otherwise causes a violation of the third rule). If you are uncertain of how this works, try to create a workable solution containing two F’s, two J’s, and one I. It cannot be done.
(The complete setup for this game can be found here: lsat/viewtopic.php?t=2035)
The correct answer choice is (D)
If G does not work during the week, then from the last rule we know that H cannot work during the week. Hence, only F, I, and J can work, and the only available numerical distribution is the 2-2-1. Despite this powerful information, and the application of the fourth rule, this is still a difficult question. If you make some good inferences, but still do not see an answer choice that is immediately and obviously correct, try to use hypotheticals to solve the problem. In this instance, answer choices (A) and (C) can both be eliminated with the following hypothetical:
Note that a savvy student could use the hypothetical created in question #7 answer choice (A)—which is the same as the hypothetical above—to eliminate answer choices (A) and (C) in question #11 (remember, question #7 is a Cannot Be True question, so the four wrong answer choices are viable solutions).
Answer choices (B) and (E) can both be eliminated with the following hypothetical:
Answer choice (D) is thus proven correct by process of elimination.
After reviewing the above information, some students still wonder why it must be that I works twice during the work week. If I only works once and F and J each work twice, then F and J are forced to “overlap” in a way that violates the fourth rule (to do otherwise causes a violation of the third rule). If you are uncertain of how this works, try to create a workable solution containing two F’s, two J’s, and one I. It cannot be done.
You do not have the required permissions to view the files attached to this post.
Dave Killoran
PowerScore Test Preparation
Follow me on X/Twitter at http://twitter.com/DaveKilloran
My LSAT Articles: http://blog.powerscore.com/lsat/author/dave-killoran
PowerScore Podcast: http://www.powerscore.com/lsat/podcast/
PowerScore Test Preparation
Follow me on X/Twitter at http://twitter.com/DaveKilloran
My LSAT Articles: http://blog.powerscore.com/lsat/author/dave-killoran
PowerScore Podcast: http://www.powerscore.com/lsat/podcast/